0
$\begingroup$

I started working on this model you can see below. I noticed there are these weird flickering shadows. So I tried googling it, but I didn't find anything (I googled "Blender Flickering shadows on model") I also tried to switch my render engine to Eevee or Cycles but that didnt help either, they are still there, just less visible. Other than that I did not try anything else, so any help would be appreciated. I am using Blender 2.92

Thank you for any answer enter image description here

$\endgroup$
4
  • $\begingroup$ hello, please share one of your object: pasteall.org/blend $\endgroup$
    – moonboots
    Mar 23, 2021 at 10:03
  • 1
    $\begingroup$ @moonboots If I may inconvenience you for moment, take a brief look at blender.meta.stackexchange.com/questions/2793/… $\endgroup$ Mar 23, 2021 at 10:55
  • $\begingroup$ Hello Duarte, I understand but actually I only ask when it seems (to me) too hard to guess without any file, in the present case I didn't guess that it could be the clipping start, and I couldn't guess that someone will find without any file ;) $\endgroup$
    – moonboots
    Mar 23, 2021 at 11:11
  • $\begingroup$ No problem, just thought it may be useful in the future. Keep it up ;) $\endgroup$ Mar 23, 2021 at 11:26

1 Answer 1

3
$\begingroup$

Looks like you have view clipping issues. In your properties panel on the right-hand side of the viewport (N) you should see a 'View' section. I suspect you have set the 'Clip Start' and/or 'Clip End' to an extreme value. Try adjusting those to something realistic to your scene (eg, 0.001m Start, 1000m End).

Blender uses the Clip Start and Clip End in calculating distances but only has a limited precision for holding and calculating values. If you use extreme ranges Blender is unable to calculate intersections and edges accurately and you see artifacts such as what you are seeing. By choosing a more reasonable range Blender can apply suitable precision and the artifacts are no longer visible.

$\endgroup$
1
  • 1
    $\begingroup$ That helped! My values were as you said, so I highered the Start to 1m and it worked, thank you for your help. $\endgroup$ Mar 23, 2021 at 10:31

Not the answer you're looking for? Browse other questions tagged .